Đến nội dung

mathandyou nội dung

Có 164 mục bởi mathandyou (Tìm giới hạn từ 30-03-2020)



Sắp theo                Sắp xếp  

#663742 Tìm nguyên hàm

Đã gửi bởi mathandyou on 04-12-2016 - 00:17 trong Tích phân - Nguyên hàm

 

Tìm nguyên hàm của:
a) $\int \frac{x+1}{\sqrt{x^2+6x+9}}$
b) $\int \frac{1}{a \; cos \; x +b}$ với $b>a>0$

 

có nhầm lẫn bạn ơi. xin lỗi nhé




#663599 Tìm nguyên hàm

Đã gửi bởi mathandyou on 01-12-2016 - 23:53 trong Tích phân - Nguyên hàm

Tìm nguyên hàm của:

a) $\int \frac{x+1}{\sqrt{x^2+6x+13}}$

b) $\int \frac{1}{a \; cos \; x +b}$ với $b>a>0$




#524685 Đề thi khảo sát đội tuyển toán Chuyên Yên Bái

Đã gửi bởi mathandyou on 15-09-2014 - 19:16 trong Thi HSG cấp Tỉnh, Thành phố. Olympic 30-4. Đề thi và kiểm tra đội tuyển các cấp.

Bài hìnhhttp://cuoichutdi.wo...rtlist-2012-g3/




#511230 Chứng minh rằng đường thẳng Euler của tam giác ABC đi qua D

Đã gửi bởi mathandyou on 06-07-2014 - 18:21 trong Hình học

http://forum.mathsco...?t=46791&page=2
Bài 10 đấy bạn.
Mở rộng của nó:

Cho tam giác ABC, một đường tròn đi qua B và C cắt các cạnh AB, AC lần lượt tại C’ và B’. Gọi D là giao điểm của BB’ và CC’, H và H’ lần lượt là trực tâm của tam giác ABC và AB’C’. Chứng minh rằng H, H’, D thẳng hàng.




#511123 Chứng minh rằng đường thẳng Euler của tam giác ABC đi qua D

Đã gửi bởi mathandyou on 06-07-2014 - 07:15 trong Hình học

http://forum.mathsco...ead.php?t=46791
bạn xem tại đây,bài 9 bài 10 gì đó :D




#510326 Chứng minh rằng đường tròn ngoại tiếp tam giác PKL luôn đi qua điểm cố định k...

Đã gửi bởi mathandyou on 02-07-2014 - 15:41 trong Hình học

Đây là lời giải cùa một bạn trên AoPS

Gọi $X,Y$ lần lượt là trung điểm $AB,AC$.

Ta chứng minh được $ \triangle AKX\sim\triangle LAY $ sau đó chứng minh $ \triangle MXK\sim\triangle LYM $.

Từ đó

$ \angle KML=\angle (KM,XM)+\angle (MY,ML)+\angle (MY,MX)=180-\angle KXM+\angle A=180-(90+\angle A)+\angle A=90 $




#504300 CMR tồn tại một tam giác có chu vi nhỏ hơn $2013$

Đã gửi bởi mathandyou on 05-06-2014 - 20:57 trong Tổ hợp và rời rạc

Cứ $3$ điểm bất kì ta có $1$ tam giác.Tô màu cạnh nhỏ nhất của các tam giác đó bằng màu đỏ.Suy ra cạnh được tô màu có độ dài nhỏ hơn $671$Ta sẽ đi chứng minh tồn tại một ta giác có $3$ cạnh màu đỏ.

Chứng minh đó bạn xem ở đây:http://cuoichutdi.wo...ai-toan-to-hop/

Ta có ngay đpcm




#494029 $P(x^2+1)=(P(x))^2+1$

Đã gửi bởi mathandyou on 20-04-2014 - 08:07 trong Đa thức

Bài này là bài 73 trong cuốn TLCT 12 phần chuyên đề về phương trình hàm đa thức




#493903 $x^2+y^2+z^2=3$.

Đã gửi bởi mathandyou on 19-04-2014 - 17:50 trong Bất đẳng thức và cực trị

Cho $x,y,z>0$ thỏa $x^2+y^2+z^2=3$.Chứng minh rằng:

$\frac{1}{{{\left( {{x}^{2}}+{{y}^{2}} \right)}^{2}}}+\frac{1}{{{\left( {{y}^{2}}+{{z}^{2}} \right)}^{2}}}+\frac{1}{{{\left( {{x}^{2}}+{{z}^{2}} \right)}^{2}}}\ge \frac{3}{32}xyz({{x}^{2}}+{{y}^{2}})({{y}^{2}}+{{z}^{2}})({{x}^{2}}+{{z}^{2}})$




#492933 Toppic Các bài toán BĐT qua các kì thi olympic 30/4

Đã gửi bởi mathandyou on 14-04-2014 - 20:39 trong Bất đẳng thức - Cực trị

Bài 3: Olympic 30-4-2014:

Cho các số thực dương $a,b,c$. Chứng minh :

$\dfrac{a}{\sqrt{7a^2+b^2+c^2}}+\dfrac{b}{\sqrt{a^2+7b^2+c^2}}+\dfrac{c}{\sqrt{a^2+b^2+7c^2}}\leq 1$



#491584 Korean NMO 2014

Đã gửi bởi mathandyou on 09-04-2014 - 10:04 trong Thi HSG Quốc gia và Quốc tế

 

Ngày 2 (23/03/2014)
Câu 4
Cho tam giác cân $ABC$ có $AC=BC$. Gọi $G$ là một điểm nằm trên $BA$ sao cho $A$ nằm giữa $B$ và $D$. Gọi $(O_1)$ là đường tròn ngoại tiếp tam giác $DAC$, $E$ là giao điểm của $(O_1)$ với $BC$. Gọi $F$ là một điểm trên $BC$ sao cho $FD$ là tiếp tuyến của $(O_1)$; gọi $(O_2)$ là đường tròn ngoại tiếp tam giác $DBF$. Hai đường tròn $(O_1),(O_2)$ cắt nhau ở điểm thứ hai $G$. Gọi $(O)$ là đường tròn ngoại tiếp tam giác $BEG$. Chứng minh rằng $FG$ là tiếp tuyến của đường tròn $(O)$ khi và chỉ khi $DG \perp FO$.
 
 

Korea 2014 prb 4.jpg

Ta có:$\angle CAB=\angle DEB$ nên $\triangle DBE \sim CBA$.

Suy ra:$DB=DE$ nên $DO$ là đường trung trực của $BE$.

Ta lại có:tứ giác $DBFD$ nội tiếp và $FD$ là tiếp tuyến của $(O_1)$ nên:

$\angle EBG=\angle BDF=\angle DEG$

Suy ra: $DE$ là tiếp tuyến của $(O)$.

Do đó:$ OG^2=OE^2=OM.OD\Rightarrow $ $\triangle OMG\sim \triangle OGD$

$\Rightarrow\angle OGM=\angle ODG $

Vì $ \angle OMF=90 $ nên $FG$ là tiếp tuyến của $(O)$ khi và chỉ khi: $ \angle OGF=90\iff O,M,G,F $

$ \iff\angle OFM=\angle MGO=\angle ODG\iff DF\perp OF $

Bài ngày một giông giống mà nhìn hình loằng ngoằn quá.TT

http://cuoichutdi.wo...2014-problem-4/




#490589 $abc(a+b+c) +(a^2+b^2+c^2)^2 \geq 4\sqrt{3(a^2+b^2+c^2...

Đã gửi bởi mathandyou on 04-04-2014 - 16:25 trong Bất đẳng thức và cực trị

Cho $a,b,c>0$.Chứng minh:$abc(a+b+c) +(a^2+b^2+c^2)^2 \geq 4\sqrt{3(a^2+b^2+c^2)}abc$




#490553 Topic ôn thi HSG lớp 10 Đồng Bằng Bắc Bộ và Olympic 30-4

Đã gửi bởi mathandyou on 04-04-2014 - 10:37 trong Thi HSG cấp Tỉnh, Thành phố. Olympic 30-4. Đề thi và kiểm tra đội tuyển các cấp.

Ủng hộ hai bài mai thi rồi. :D

Bài 28:Cho $a,b,c$ là các số thực dương.Chứng minh bất đẳng thức:

$(\dfrac{1}{a}+\dfrac{1}{b}+\dfrac{1}{c})(\dfrac{1}{a+1}+\dfrac{1}{b+1}+\dfrac{1}{c+1}) \geq \dfrac{9}{1+abc}$

Bài 29:Cho các số thực $a_1,a_2,..,a_n$ đồng thời thỏa mãn:$a_1+a_2+..+a_n \geq n$ và $a_1^2+a_2^2+..+a_n^2 \geq n^2$.

Chứng minh: $max$ {a_1,a_2,..,a_n} $\geq 2$




#488200 CMR: BD là tiếp tuyến của đường tròn ngoại tiếp tam giác ADZ

Đã gửi bởi mathandyou on 22-03-2014 - 10:51 trong Hình học

Cho tam giác ABC vuông tại A và B<C. Tiếp tuyến tại A của đường tròn ngoại tiếp tam giác ABC cắt BC tại D. Gọi E là điểm đối xứng của A qua BC. X là chân đường cao hạ từ A xuống BE, Y là trung điểm AX. BY cắt đường tròn nói trên tại điểm thứ hai Z. CMR: BD là tiếp tuyến của đường tròn ngoại tiếp tam giác ADZ

bạn xem lại,hình như ghi đề bị nhầm




#488010 CMR: Tam giác ANP cân

Đã gửi bởi mathandyou on 20-03-2014 - 22:10 trong Hình học

Cho tam giác ABC vuông tại A, tâm nội tiếp I. (I) tiếp xúc với BC, CA, AB tại D, E, F. Gọi M là trung điểm AC, Mi cắt AB tại N, DF cắt đường cao AH tại P. CMR: Tam giác ANP cân

http://forum.mathsco...ead.php?t=46921




#487942 $P=9[x^2(y-z)^2+y^2(z-x)^2+z^2(x-y)^2]+18(xy+yz+zx).$

Đã gửi bởi mathandyou on 20-03-2014 - 15:52 trong Bất đẳng thức và cực trị

Cho $x,y,z\in\mathbb{R}$ thỏa mãn $x^2+y^2+z^2=9.$
Tìm giá trị lớn nhất của 
$P=9[x^2(y-z)^2+y^2(z-x)^2+z^2(x-y)^2]+18(xy+yz+zx).$



#487135 GPT: $2^{x^{5}}+4^{x^{4}}+256^...

Đã gửi bởi mathandyou on 16-03-2014 - 11:40 trong Phương trình - Hệ phương trình - Bất phương trình

Lời giải :

Theo BĐT $AM-GM$ :

$$2^{x^5}+4^{x^4}+256^4\geq 3\sqrt[3]{2^{x^5}.4^{x^4}.256^4}=3\sqrt[3]{2^{x^5+2x^4+32}}\geq 3\sqrt[3]{2^{3\sqrt[3]{x^5.2x^4.32}}}=3\sqrt[3]{2^{12.x^3}}=3.16^{x^3}$$

Dấu bằng phải xảy ra khi và chỉ khi $$\left\{\begin{matrix} 2^{x^5}=4^{x^4}=256^4\\ x^5=2x^4=32 \end{matrix}\right.\Leftrightarrow x=2$$

Nghiệm của phương trình là $x=2$

x>0 không chú?




#486654 CĐT Olympic 30-4 toán 10 (lần 3) THPT Chuyên Lương Thế Vinh, Đồng Nai

Đã gửi bởi mathandyou on 13-03-2014 - 18:49 trong Thi HSG cấp Tỉnh, Thành phố. Olympic 30-4. Đề thi và kiểm tra đội tuyển các cấp.

Bài 1: $\sqrt{7x^{2}+25x+19}-\sqrt{x^{2}-2x-35}=7\sqrt{x+2} <=>(\sqrt{x^{2}-2x-35}+7\sqrt{x+2})^{2}=7x^{2}+25x+19$

$<=>3x^{2}-11x-22=7\sqrt{(x^{2}-5x-14)(x+5)}$

Đặt $\left\{\begin{matrix} x^{2}-5x-14=u\\ x+5=v \end{matrix}\right.$

Ta được phương trình: $3u+4v=7\sqrt{uv}<=>\left\{\begin{matrix} 3u+4v\geq 0\\ \begin{bmatrix} u=v\\ u=\frac{16}{9}v \end{bmatrix} \end{matrix}\right.$

$<=>\begin{bmatrix} x^{2}-5x-14=x+5\\ x^{2}-5x-14=\frac{16}{9}(x+5) \end{bmatrix}<=>\begin{bmatrix} x=3+2\sqrt{7}\\ x=3-2\sqrt{7} \\ x=\frac{61+\sqrt{11137}}{18} \\ x=\frac{61-\sqrt{11137}}{18} \end{bmatrix}$

Thử lại: $x=3+2\sqrt{7};x=\frac{61+\sqrt{11137}}{18}$ là nghiệm của phương trình.

Bài 3:

Đặt $\frac{x^{2}+pxy}{xy}=k=>x^{2}-kxy+py^{2}=0=>x=\frac{k\pm \sqrt{k^{2}-4p}}{2}.y$

Giả sử $\sqrt{k^{2}-4p}\epsilon \mathbb{I}=>x=i.y$ (trong đó $i\epsilon \mathbb{I}$)

Điều này vô lý vì $x,y\epsilon Z^{+}$

Vậy $\sqrt{k^{2}-4p}=m(m\epsilon Z^{+})$$=>k^{2}-4p=m^{2}=>(k+m)(k-m)=4p$

Mà $k+m;k-m$ cùng tính chẵn, lẻ và $k+m>k-m$ nên xảy ra các trường hợp sau:

Trường hợp 1: $\left\{\begin{matrix} k+m=4\\ k-m=p \end{matrix}\right.<=>\left\{\begin{matrix} p=2\\ k=3 \\ m=1 \end{matrix}\right.=>\begin{bmatrix} x=y\\x=2y \end{bmatrix}=>\frac{x^{2}+py^{2}}{xy}=3=p+1$

Trường hợp 2: $\left\{\begin{matrix} k+m=p\\k-m=4 \end{matrix}\right.=>k=\frac{p+4}{2}=>\begin{bmatrix} x=py\\ x=y \end{bmatrix}=>\frac{x^{2}+py^{2}}{xy}=p+1$

Trường hợp 3: $\left\{\begin{matrix} k+m=2p\\k-m=2 \end{matrix}\right.=>k=p+1=>\begin{bmatrix} x=py\\ x=y \end{bmatrix}=>\frac{x^{2}+py^{2}}{xy}=p+1$

Vậy $\frac{x^{2}+py^{2}}{xy}=p+1$

Bài 4: Theo giả thiết ta có:    $f(m+f(n))=f(m)-n (1)$

Giả sử $f(a)=f(b)=>f(m+f(a))=f(m+f(b))=>f(m)-a=f(m)-b=>a=b$$=> $$f$ đơn ánh

Trong $(1)$ thay $m$ bởi $f(m)$ ta được:

$f(f(m)+f(n))=f(f(m))-n=f(0+f(m))-n=f(0)-(m+n)=f(f(m+n))=>f(m)+f(n)=f(m+n)$$=>f(n)=an$

Thay $=>f(n)=an$ vào $(1)$ ta được: $a(m+an)=am-n=>a^{2}=-1$ (vô lý)

Vậy không tồn tại $f$ thỏa

Bài 5:

Ta nhận thấy $a_{m}+a_{n}\equiv a_{m}-a_{n}\equiv a_{n}-a_{m}(mod 2)$

Vì vậy sau $k$ lần thay số thì tồng các số của dãy mới cùng tính chẵn lẻ với dãy ban đầu

Mà tồng các số của dãy ban đầu là: $1+2+3+...+2014\equiv 2029105\equiv 1(mod2)$

Suy ra sau $2013$ lần thay số thì còn lại moật số và dố đó phải là số lẻ

Vì vậy sau $2013$ lần thay số thì còn một số và số đó khác 0.

woa  :ohmy:




#485297 $xf(y)+yf(x)=(x+y)f(x^2+y^2),\;\forall x,y\in \mathb...

Đã gửi bởi mathandyou on 01-03-2014 - 19:52 trong Phương trình hàm

-Với $x=y=0= > f(0)=0$

-Với $x=0= > yf(0)=yf(y^2)< = > y(f(0)-f(y^2))=0$

-Với $y=0$.Từ pt ban đầu $= > xf(0)+0f(x)=xf(x^2)< = > xf(x^2)=0< = > x=0,f(x^2)=0= > f(x)=0$

-Với $f(0)=f(y^2)= > y=0= > f(x)=0$

Hàng đầu là sai rồi né.

Đây là trên tập số tự nhiên nhé bạn.$f(x^2)=0$ không suy ra được $f(x)=0$




#485163 $f: \mathbb{N} \rightarrow \mathbb{N}...

Đã gửi bởi mathandyou on 28-02-2014 - 19:52 trong Phương trình hàm

Bài này trên R vẫn đúng.Chắc bạn chưa học cách CM PTH Cauchy trên R nên mới cho trên N để cm qui nạp ra.:)




#484830 $B,I_1,I_2.N$ đồng viên

Đã gửi bởi mathandyou on 25-02-2014 - 21:32 trong Hình học

Cho tam giác $ABC$ ngoại tiếp $(O)$.$N$ là trung điểm cung $ABC$.Gọi $M$ là trung điểm $BC$.Gọi $I_1,I_2$ lần lượt là tâm đường tròn nội tiếp tam giác $ABM$ và $ACM$.Chứng minh:$B,I_1,I_2.N$ đồng viên




#484646 $ab+bc+ca=3$

Đã gửi bởi mathandyou on 24-02-2014 - 20:04 trong Bất đẳng thức - Cực trị

Đặt vế trái bất đẳng thức là $P$

Áp dụng Cauchy-Schwarzt ta có 

         $P^2\leqslant (a+b+c)(\frac{ab+ac}{a^2+bc}+\frac{ba+bc}{b^2+ac}+\frac{ca+cb}{c^2+ab})$

Bây giờ ta sẽ chứng minh $\frac{ab+ac}{a^2+bc}+\frac{ba+bc}{b^2+ac}+\frac{ca+cb}{c^2+ab}\leqslant 3$

       $\Leftrightarrow \frac{(a-b)(a-c)}{a^2+bc}+\frac{(b-a)(b-c)}{b^2+ac}+\frac{(c-a)(c-b)}{c^2+ab}\geqslant 0$

Luôn đúng theo Schur suy rộng

Vậy $P^2\leqslant 3(a+b+c)$

Ta chỉ cần chứng minh $3(a+b+c)\leqslant \frac{9}{(abc)^2}\Leftrightarrow abc(a+b+c).abc\leqslant 3$

Áp dụng AM-GM ta có $\left\{\begin{matrix} (ab+bc+ca)^2 \geqslant 3abc(a+b+c)\\ab+bc+ca\geqslant 3\sqrt[3]{(abc)^2} \end{matrix}\right.$

                      $\Rightarrow abc(a+b+c).abc \leqslant 3$

Vậy ta có đpcm

Đẳng thức xảy ra khi $a=b=c=1$

 

Chỗ màu đỏ là $a^2+b^2+c^2$ phải không ạ?




#484619 $ab+bc+ca=3$

Đã gửi bởi mathandyou on 24-02-2014 - 18:59 trong Bất đẳng thức - Cực trị

Cho $a,b,c>0$ và thỏa:$ab+bc+ca=3$.Chứng minh rằng:

$$a\sqrt{\frac{b+c}{a^2+bc}}+b\sqrt{\frac{c+a}{b^2+ca}}+c\sqrt{\frac{a+b}{c^2+ab}} \leq \frac{3}{abc}$$




#484615 chọn đội dự tuyển THPT Chuyên Quốc Học ngày 2 2013-2014

Đã gửi bởi mathandyou on 24-02-2014 - 18:52 trong Thi HSG cấp Tỉnh, Thành phố. Olympic 30-4. Đề thi và kiểm tra đội tuyển các cấp.

Bài 2:tổng quát là định lí sivestre.Chứng minh bằng qui nạp.Cũng từng là đề thi hsg QG




#484558 Chọn đội tuyển HSG lớp 10 THPT Chuyên Lương Thế Vinh, Đồng Nai 2013-2014

Đã gửi bởi mathandyou on 24-02-2014 - 12:28 trong Thi HSG cấp Tỉnh, Thành phố. Olympic 30-4. Đề thi và kiểm tra đội tuyển các cấp.

Ví dụ p = 2, q = 5, n = 4 thì $ 5. (5 + 1) = 2. 15 = 2( 2^{3} + 2^{2} + 2 + 1) $ (thỏa mãn)

$\Rightarrow 5 \vdots 15$ hoặc $6 \vdots 15$ ???

Đó là hướng giải thôi bạn.Từ những điều đó chỉ ra mâu thuẫn rồi đi đến lời giải.